25
A Guide to Logical Reasoning Questions What Is an Argument? Most Logical Reasoning questions focus on arguments, which are sets of statements that present evidence and draw a conclusion on the basis of that evidence. These arguments are generally short and self-contained. Consider this basic example: Sarah is well-qualified, and the hiring committee is very familiar with her work. Therefore, she will probably receive a job offer. This is a simple argument. Two pieces of evidence are presented. These are the premises of the argument. These premises are offered in support of the view that Sarah will probably receive a job offer. This is the conclusion of the argument. Let’s look at a second case: Computer Whiz is a well-respected magazine with a large readership, so its product endorsements carry a lot of weight in the computer electronics marketplace. The X2000 display monitor was recently endorsed by Com- puter Whiz. It is therefore likely that sales of the X2000 monitor will increase dramatically. In this argument, information about the magazine’s reputation and large readership serves as a basis for reaching an intermediate, or subsidiary , conclusion: that its endorsements are very influential in the market- place. This intermediate conclusion in conjunction with a premise that reports that the X2000 was recently endorsed by the magazine provides the grounds for the prediction of an increase in sales. This prediction is the main, or overall, conclusion of the argument. n Identifying the Parts of an Argument An argument can be analyzed by identifying its various parts and the roles that those parts play. The most basic parts of an argument are premises and conclusions. As we have already seen, an argument may have one or more interme- diate conclusions in addition to its overall conclusion. Premises come in a variety of forms. Some premises are specific matters of fact, some are definitions, and others are broad principles or generalizations. What all premises have in common is that they are put forward as true with- out support. That is, there is no attempt within the argument to prove or justify them. In contrast, a conclu- sion is not simply asserted. A conclusion is presented as being justified by certain premises. Thus, the conclusion of an argument is open to the challenge that it is not ade- quately supported by the premises. (Premises, of course, can also be challenged, on grounds such as factual accu- racy, but such challenges are not matters of logic.) One thing to remember about premises and conclu- sions is that they can come in any order. Premises are presented in support of a conclusion, but this does not mean that premises always precede the conclusion. A conclusion may be at the beginning, middle, or end of an argument. Consider the following examples: Dolores is far more skillful than Victor is at securing the kind of financial support the Volunteers for Literacy Program needs, and Dolores does not have Victor’s propensity for alienating the program’s most dedicated volunteers. There- fore, the Volunteers for Literacy Program would benefit if Dolores took Victor’s place as director. Dolores is far more skillful than Victor is at securing the kind of financial support the Volunteers for Literacy Pro- gram needs. Therefore, the program would benefit if Dolores took Victor’s place as director, especially since Dolores does not have Victor’s propensity for alienating the program’s most dedicated volunteers. The Volunteers for Literacy Program would benefit if Dolores takes Victor’s place as director, since Dolores is far more skillful than Victor is at securing the kind of financial support the program needs and Dolores does not have Vic- tor’s propensity for alienating the program’s most dedicated volunteers. [September 1995 LSAT, section 2, question 19] These three examples all present the same argument. In each example, the conclusion is that the Volunteers for Literacy Program would benefit if Dolores took Victor’s place as director, and this conclusion is supported by the same two premises. But each example expresses the argument in a different way, with the conclusion appear- ing in the final, middle, and initial position, respectively. It is important, then, to focus on the role each statement plays in the argument as a whole. Position within the argument simply doesn’t matter. Another thing to keep in mind is the presence of indicator words that mark the roles that statements play in argu- ments. For example, “therefore” often precedes a conclusion; it is a common conclusion indicator. So are “thus,” “hence,” “consequently,” “it follows that,” “it can be concluded that,” and various others. Similarly, premises are often preceded by indicator words, the most typical being “since” and “because.” However, do not rely uncriti- cally on these indicator words. They can be misleading, especially in the case of complex arguments, which might contain one or more subarguments. There is no completely mechanical way of identifying the roles that various state- ments play within an argument. It is worth noting that people, in making arguments, often do not confine themselves to presenting just the con- clusion and the statements that support it. Likewise, the short arguments in Logical Reasoning questions often include statements that are neither premises nor conclu- sions. This includes statements that indicate the motivation for making the argument, statements that convey back- ground information, and statements that identify the 15

A Guide to Logical Reasoning Questions · A Guide to Logical Reasoning Questions What Is an Argument? Most Logical Reasoning questions focus on arguments, which …

Embed Size (px)

Citation preview

A Guide to Logical Reasoning Questions

What Is an Argument?

Most Logical Reasoning questions focus on arguments,which are sets of statements that present evidence anddraw a conclusion on the basis of that evidence. Thesearguments are generally short and self-contained.

Consider this basic example:

Sarah is well-qualified, and the hiring committee is veryfamiliar with her work. Therefore, she will probably receivea job offer.

This is a simple argument. Two pieces of evidence arepresented. These are the premises of the argument.These premises are offered in support of the view thatSarah will probably receive a job offer. This is theconclusion of the argument.

Let’s look at a second case:

Computer Whiz is a well-respected magazine with alarge readership, so its product endorsements carry a lot ofweight in the computer electronics marketplace. TheX2000 display monitor was recently endorsed by Com-puter Whiz. It is therefore likely that sales of the X2000monitor will increase dramatically.

In this argument, information about the magazine’sreputation and large readership serves as a basis forreaching an intermediate, or subsidiary, conclusion:that its endorsements are very influential in the market-place. This intermediate conclusion in conjunction with apremise that reports that the X2000 was recentlyendorsed by the magazine provides the grounds for theprediction of an increase in sales. This prediction is themain, or overall, conclusion of the argument.

� Identifying the Parts of an Argument

An argument can be analyzed by identifying its variousparts and the roles that those parts play. The most basic partsof an argument are premises and conclusions. As we havealready seen, an argument may have one or more interme-diate conclusions in addition to its overall conclusion.

Premises come in a variety of forms. Some premises arespecific matters of fact, some are definitions, and othersare broad principles or generalizations. What all premiseshave in common is that they are put forward as true with-out support. That is, there is no attempt within theargument to prove or justify them. In contrast, a conclu-sion is not simply asserted. A conclusion is presented asbeing justified by certain premises. Thus, the conclusionof an argument is open to the challenge that it is not ade-quately supported by the premises. (Premises, of course,can also be challenged, on grounds such as factual accu-racy, but such challenges are not matters of logic.)

One thing to remember about premises and conclu-sions is that they can come in any order. Premises are

presented in support of a conclusion, but this does notmean that premises always precede the conclusion. Aconclusion may be at the beginning, middle, or end of anargument. Consider the following examples:

Dolores is far more skillful than Victor is at securing the kindof financial support the Volunteers for Literacy Programneeds, and Dolores does not have Victor’s propensity foralienating the program’s most dedicated volunteers. There-fore, the Volunteers for Literacy Program would benefit ifDolores took Victor’s place as director.

Dolores is far more skillful than Victor is at securing thekind of financial support the Volunteers for Literacy Pro-gram needs. Therefore, the program would benefit ifDolores took Victor’s place as director, especially sinceDolores does not have Victor’s propensity for alienatingthe program’s most dedicated volunteers.

The Volunteers for Literacy Program would benefit ifDolores takes Victor’s place as director, since Dolores is farmore skillful than Victor is at securing the kind of financialsupport the program needs and Dolores does not have Vic-tor’s propensity for alienating the program’s most dedicatedvolunteers. [September 1995 LSAT, section 2, question 19]

These three examples all present the same argument.In each example, the conclusion is that the Volunteers forLiteracy Program would benefit if Dolores took Victor’splace as director, and this conclusion is supported by thesame two premises. But each example expresses theargument in a different way, with the conclusion appear-ing in the final, middle, and initial position, respectively.It is important, then, to focus on the role each statementplays in the argument as a whole. Position within theargument simply doesn’t matter.

Another thing to keep in mind is the presence of indicatorwords that mark the roles that statements play in argu-ments. For example, “therefore” often precedes aconclusion; it is a common conclusion indicator. So are“thus,” “hence,” “consequently,” “it follows that,” “it canbe concluded that,” and various others. Similarly, premisesare often preceded by indicator words, the most typicalbeing “since” and “because.” However, do not rely uncriti-cally on these indicator words. They can be misleading,especially in the case of complex arguments, which mightcontain one or more subarguments. There is no completelymechanical way of identifying the roles that various state-ments play within an argument.

It is worth noting that people, in making arguments,often do not confine themselves to presenting just the con-clusion and the statements that support it. Likewise, theshort arguments in Logical Reasoning questions ofteninclude statements that are neither premises nor conclu-sions. This includes statements that indicate the motivationfor making the argument, statements that convey back-ground information, and statements that identify the

15

position the argument comes out against. So don’t assumethat everything that is not part of the argument’s conclu-sion must be functioning as support for that conclusion.

� How the Argument Goes

Once you have identified the premises and the conclu-sion, the next step is to get clear about exactly how theargument is meant to go; that is, how the grounds offeredfor the conclusion are actually supposed to bear on the con-clusion. Understanding how an argument goes is a crucialstep in answering many questions that appear on the LSAT.This includes questions that ask you to identify a reasoningtechnique used within an argument, questions that requireyou to match the pattern of reasoning used in two separatearguments, and of a variety of other question types.

Determining how an argument goes involves discerninghow the premises are supposed to support the overall con-clusion. Consider, for example, the argument presentedearlier about the Volunteers for Literacy Program, whichconcludes that the program would benefit if Dolores tookVictor’s place as director. Two considerations in support ofthis conclusion are offered: one asserting Dolores’s superi-ority in securing financial support and the other chargingthat Victor is more prone to alienating dedicated volun-teers. These two considerations are both relevant to theconclusion since, all other things being equal, a programbenefits from having a director who is both better atfund-raising and less likely to alienate volunteers. Each ofthese considerations provides some support for the conclu-sion, and the support provided by one is completelyindependent of the support provided by the other.

In other arguments, the way in which premises sup-port a conclusion can be much more complex. Considerthis example:

The years 1917, 1937, 1956, 1968, 1979, and 1990 are allnotable for the occurrence of both popular uprisings andnear-maximum sunspot activity. During heavy sunspotactivity, there is a sharp rise in positively charged ions inthe air people breathe, and positively charged ions areknown to make people anxious and irritable. Therefore, itis likely that sunspot activity has actually been a factor intriggering popular uprisings.2

The conclusion of this argument, signaled by “There-fore,” is that it is likely that sunspot activity has been afactor in triggering popular uprisings. There are three pre-mises. The first tells us about specific years in which bothheavy sunspot activity and popular uprisings occurred.The other two are generalizations: that there is a sharp rise

in positively charged ions in the air during heavy sun-spot activity, and that positively charged ions makepeople anxious and irritable.

So how does this argument go? The first premise pro-vides some direct support for the conclusion, but thissupport is very weak, circumstantial evidence. The secondand third premises do not support the conclusion directly,but only in conjunction with each other. If these twopremises are true, they work together to establish thatsunspots are a causal factor in increased irritability. Noticethat there is still no link between sunspots and popularuprisings. There is some plausibility, however, to the ideathat increased irritability makes popular uprisings morelikely, and the argument tacitly assumes that this is in fact so.

If we make this assumption then, we can see theconnection between sunspot activity and popularuprisings. This greatly enhances the evidence that thefirst premise provides.

Cross-reference: You can learn more about therole of assumptions in arguments in thediscussion starting on pages 28–31.

� Questions About How the Argument Goes

Your test may include questions that ask you about howan argument proceeds overall, or about the logical roleplayed by a particular part of an argument, or about thelogical move one participant in a dialogue makes inresponding to the other. Understanding how the relevantargument goes puts you in a position to answer thesequestions. Three examples are briefly discussed below.

Example 1

Red squirrels are known to make holes in the bark of sugarmaple trees and to consume the trees’ sap. Since sugarmaple sap is essentially water with a small concentrationof sugar, the squirrels almost certainly are after eitherwater or sugar. Water is easily available from othersources in places where maple trees grow, so the squirrelswould not go to the trouble of chewing holes in trees justto get water. Therefore, they are probably after the sugar.3

The question based on this argument is simply:

The argument proceeds by

A Guide to Logical Reasoning Questions16

2 February 1995 LSAT, section 4, question 8.3 June 2002 LSAT, section 4, question 10.

The conclusion of this argument is quite easy to identify:red squirrels, in making holes in the bark of sugar mapletrees, are probably after the sugar contained in the trees’sap. The argument arrives at this conclusion by first notingthat since maple tree sap is essentially just water and sugar,the squirrels must be after either the one or the other. Theargument goes on to reject the idea that it is the water thatthe squirrels are after, on the grounds that water is readilyavailable for less effort where maple trees grow.

Once you have figured out how the argument goes,you’re ready to check the answer choices to find the bestcharacterization of the argument’s reasoning. In this par-ticular case, the best characterization is:

rejecting a possible alternative explanation for anobserved phenomenon

This is not the only way to describe how the argumentproceeds, and it may not be the description you wouldhave given. But it is an accurate characterization and isthus the correct answer. So keep in mind when checkingthe answer choices that the correct answer may be justone of several acceptable ways of putting things.

Example 2

In order to determine automobile insurance premiums fora driver, insurance companies calculate various risk fac-tors; as the risk factors increase, so does the premium.Certain factors, such as the driver’s age and past accidenthistory, play an important role in these calculations. Yetthese premiums should also increase with the frequencywith which a person drives. After all, a person’s chance ofbeing involved in a mishap increases in proportion to thenumber of times that person drives.4

The question based on this argument is:

The claim that insurance premiums should increase as thefrequency with which a driver drives increases playswhich one of the following roles in the argument?

The first step in determining how this argument goesis identifying the conclusion. To do this, find the positionfor which the argument offers support.

The short phrase “after all” at the beginning of thefourth sentence indicates that the statement that followsfunctions as a premise. This premise essentially says thatthe frequency with which a person drives is a factor intheir risk of being involved in a traffic accident. Weknow from the first sentence that risk factors matter in

determining a driver’s automobile insurance premiums:as certain risk factors increase, the premium increases.Putting all of this together, we see that the argument isconstructed to support the position stated in the thirdsentence: “…these premiums should also increase withthe frequency with which the person drives.”

So the claim that insurance premiums should increaseas the frequency with which a driver drives increases isthe conclusion of the argument. That is its role in theargument. The answer choice that expresses this, in someway or other, is the correct one.

Example 3

Zachary: The term “fresco” refers to paint that has beenapplied to wet plaster. Once dried, a fresco indelibly preservesthe paint that a painter has applied in this way. Unfortu-nately, additions known to have been made by later paintershave obscured the original fresco work done by Michelangeloin the Sistine Chapel. Therefore, in order to restore Michel-angelo’s Sistine Chapel paintings to the appearance thatMichelangelo intended them to have, everything exceptthe original fresco work must be stripped away.

Stephen: But it was extremely common for painters ofMichelangelo’s era to add painted details to their ownfresco work after the frescos has dried.5

The corresponding question is:

Stephen’s response to Zachary proceeds by

Zachary tells us that Michelangelo’s frescoes in theSistine Chapel had additions made to them by laterpainters. On the basis of this he argues that everythingexcept Michelangelo’s original fresco work has to bestripped away if the paintings are to have the appearanceMichelangelo intended them to have.

Stephen’s response makes clear that for painters ofMichelangelo’s era, the frescoes as originally executed didnot necessarily have the appearance that those paintersintended them to have. So Stephen’s response points toand casts doubt on an assumption of Zachary’s argument.This assumption is that Michelangelo did not make addi-tions to his own fresco work in order to give thepaintings the appearance that he wanted them to have.

Turning to the answer choices, you find this statementamong them:

calling into question an assumption on which Zachary’sconclusion depends

A Guide to Logical Reasoning Questions 17

4 October 2002 LSAT, section 1, question 18.5 October 2001 LSAT, section 4, question 25.

This statement correctly characterizes how Stephen’sresponse to Zachary proceeds.

A Point to Consider

� Arguments vary widely in their strength, that is, inthe extent to which their conclusions are justified bytheir premises. In the extreme case—the case of a“deductively valid” (i.e., conclusive) argument—thetruth of the conclusion is completely guaranteed by thetruth of the premises. In other words, anyone whoaccepts those premises is thereby committed to accept-ing the conclusion. In most cases, however, therelationship of the premises to the conclusion is lessstrict: the premises provide some grounds for acceptingthe conclusion, but these grounds are not airtight. Inother words, someone might accept all of the premisesof such an argument yet still be logically justified in notaccepting its conclusion.

� Identifying the Main Conclusion of an Argument

Some questions present you with an argument and askyou to identify its main conclusion. In questions of thiskind, the conclusion is actually drawn in the argument,but it is often stated somewhat indirectly and it is some-times not signaled by any of the standard conclusion—indicator words such as “therefore” or “thus.” To identifythe conclusion, therefore, you also need to look at what thestatements in the argument mean, and how they are relatedto each other. Look for a position that the argument as awhole is trying to establish, and rule out any statementsthat, either directly or indirectly, give reasons for that posi-tion. You should also eliminate statements that merelyestablish a context or supply background information.

An Example

Journalist: Obviously, though some animals are purelycarnivorous, none would survive without plants. But thedependence is mutual. Many plant species would neverhave come to be had there been no animals to pollinate,fertilize, and broadcast their seeds. Also, plants’photosynthetic activity would deplete the carbon dioxidein Earth’s atmosphere were it not constantly being replen-ished by the exhalation of animals, engine fumes, andsmoke from fires, many set by human beings.6

The question asks:

Which one of the following most accurately expresses themain conclusion of the journalist’s argument?

So, how do you tackle this question? First, read theargument through. You might immediately recognize thatthe argument is of a familiar sort. The argument isdirected toward a position that has two sides to it: a verystraightforward one that is simply asserted and a lessobvious one that the argument goes to some trouble toestablish. The first sentence presents the straightforwardside of the position being argued for. The second sentencestates the entire position. The third and fourth sentencesmake the case for the less obvious side of the position.

Suppose that after reading the argument you are not sureexactly how it goes. What do you do then? It might be help-ful to go through the argument statement by statement andask about each statement in turn, “Does this statementreceive support from some other statement?” If so, thestatement is either a subsidiary conclusion drawn to sup-port the main conclusion or it is itself the main conclusion.If the statement does not receive support from anythingelse in the argument, ask whether it provides support forsome other statement. If it does, it’s a premise of the argu-ment, and whatever statement it provides support for iseither the main conclusion or a subsidiary conclusion.

In the journalist’s argument, the first statement does notreceive support from anything else that is said in the argu-ment. It does, however, provide support for the secondstatement by establishing one side of the dependence thatthe second statement refers to. So the second statement isa candidate for being the main conclusion of the argu-ment. If you go on to analyze the third and fourthstatements, you’ll find that neither receives any supportfrom anything else in the argument and that each inde-pendently supports the second statement by establishingthe other side of the mutual dependence. Since everythingelse in the argument goes toward supporting the secondstatement, it is clear that the second statement expressesthe main conclusion of the argument.

The second statement states the main conclusion in asomewhat abbreviated way in that it doesn’t spell outwhat is meant by “dependence.” But having workedthrough the argument, we can recognize that the follow-ing is an accurate statement of the conclusion:

Just as animals are dependent on plants for their survival,plants are dependent on animals for theirs.

The incorrect answer choices often restate a premise orpart of a premise. For example, the following incorrectanswer is a partial restatement of the fourth sentence ofthe journalist’s argument:

A Guide to Logical Reasoning Questions18

6 October 2002 LSAT, section 4, question 3.

Human activity is part of what prevents plants fromdepleting the oxygen in Earth’s atmosphere on whichplants and animals alike depend.

Other incorrect answer choices may state somethingthat can be inferred from statements in the argument butthat is not the argument’s main conclusion. Here is anexample of this, based on the journalist’s argument:

The chemical composition of Earth and its atmospheredepends, at least to some extent, on the existence andactivities of the animals that populate Earth.

Some Points to Consider

� If there is a “thus” or “therefore” in the argument, donot assume that these words introduce the main conclu-sion of the argument. They often indicate a subsidiaryconclusion rather than the main conclusion.

� With questions that ask you to identify the main conclu-sion, it is generally possible to form a fairly preciseidea of what the correct answer will be like beforeconsidering the answer choices. Doing so makes itpossible to select the correct answer very efficiently.You should also try to get a precise idea of the mainconclusion, because some of the incorrect answerchoices may be only slightly inaccurate. For example,if the actual conclusion is that something is likely tobe true, an incorrect answer choice may say that it isdefinitely true. This choice is incorrect because it goesbeyond the actual conclusion.

� Matching Patterns of Reasoning in Arguments

There is another kind of question that tests your abilityto determine how an argument goes. It begins with anargument and then asks you to choose one argument fromamong the answer choices that is most similar in its reason-ing to the initial (or reference) argument. The questionsthemselves are worded in a variety of ways, including:

The pattern of reasoning in which of the following argu-ments is most similar to that in the argument above?

Which one of the following arguments is most similar inits reasoning to the argument above?

You don’t need to come up with a verbal description ofthe pattern of reasoning in order to answer these ques-tions. All you need is a solid intuitive grasp of the logical

structure of the reference argument: what its conclusion is,and how the premises fit together to support the conclusion.

These questions are asking for a match in logical struc-ture, that is, the way the premises fit together to supportthe conclusion. So do not pay any attention to similarityor dissimilarity in subject matter, or to background mate-rial that is not part of the premises or the conclusion. Norshould you concern yourself with anything about the par-ticular way the argument is laid out, such as the order inwhich the premises and the conclusion are presented.

An Example

All known deposits of the mineral tanzanite are in Tanza-nia. Therefore, because Ashley collects only tanzanitestones, she is unlikely ever to collect a stone not originallyfrom Tanzania.7

The question asks:

Which one of the following is most similar in its reasoningto the argument above?

So what is the structure of the reasoning in the referenceargument? There are two premises, the one about tanzanitedeposits and the one about Ashley’s collecting habits. Andthere is a conclusion: Ashley is unlikely ever to collect astone not originally from Tanzania. Note that the conclu-sion merely says that something is unlikely, not that it willdefinitely not happen. The conclusion is probably qualifiedin this way because the premise about tanzanite depositsspeaks only about the known deposits of that mineral,thereby leaving open the possibility that there are undis-covered tanzanite deposits outside of Tanzania.

But also note that the argument is a fairly strong one. Thepremises give a reasonable basis for accepting the conclu-sion: if the premises are true, the only way in which Ashleywould ever collect a stone that is not originally from Tanza-nia is if tanzanite is someday discovered outside ofTanzania or if she begins to collect some different type ofstone in the future.

The next step is to check the answer choices and to findthe one with the same pattern of reasoning.

So let’s try this answer choice:

Frogs are the only animals known to live in the lagoon onScrag Island. The diet of the owls on Scrag Island consists ofnothing but frogs from the island. Therefore, the owls areunlikely ever to eat an animal that lives outside the lagoon.

A Guide to Logical Reasoning Questions 19

7 June 2002 LSAT, section 2, question 26.

Does this follow the same pattern of reasoning as theargument about tanzanite? The conclusion has the rightshape: it says that something is unlikely ever to happen, justas the conclusion of the reference argument does. In addi-tion, this argument, like the reference argument, has apremise that limits itself to speaking about what is known tobe true, thereby leaving open the possibility of cases unlikethose now known. Plus, the second premise is exclusionaryin nature: where the reference argument uses “only,” thisargument says “nothing but.” So there are a number of resem-blances between important parts of the two arguments.

However, whereas the reference argument is fairlystrong, this argument is seriously flawed. Notice that thetwo premises do not rule out the possibility there are frogson Scrag Island that do not live in the lagoon. So thereseems to be a strong possibility that the owls on ScragIsland eat frogs that aren’t from the lagoon. The conclusionof this argument thus receives little or no support from thepremises. If the reasoning in this argument were closelyparallel to that in the reference argument, its premiseswould provide similarly strong support for its conclusion.So this answer choice cannot be correct.

Let’s try another one of the answer choices:

The only frogs yet discovered on Scrag Island live in thelagoon. The diet of all the owls on Scrag Island consistsentirely of frogs on the island, so the owls will probablynever eat an animal that lives outside the lagoon.

Here, too, the conclusion has the right shape: it saysthat something is unlikely ever to happen. In addition,this argument has a premise that limits itself to speakingabout what is known to be the case. Plus, the secondpremise is exclusionary in nature.

In this case, the premises provide support for the con-clusion in just the same way that the premises in thereference argument do for the conclusion of that argu-ment. This argument can be paraphrased in a way that isparallel to the reference argument: All known frogs onScrag Island live in the lagoon. Scrag Island owls eatonly frogs. It is therefore unlikely that an owl on ScragIsland will ever eat an animal that does not live in thelagoon. Thus, the pattern of reasoning in the two argu-ments is essentially the same.

What Can Be Concluded From the Information Provided

Many Logical Reasoning questions test your ability todetermine what is supported by a body of available evi-dence. These questions ask you to pick one statementthat can in some way or another be inferred from theavailable evidence. So, in effect, you are asked to distin-guish between positions that are supported by theinformation that you have been given and positions thatare not supported by that information. These questionscome in a variety of forms.

� Identifying a Position That Is ConclusivelyEstablished by Information Provided

Some questions test your ability to identify what followslogically from certain evidence or information. For thesequestions, you will be presented with information thatprovides conclusive support for one of the answerchoices. Typical wordings for these questions include:

If the statements above are true, which one of the followingmust also be true?

Which one of the following logically follows from the state-ments above?

With these questions, you are looking for somethingthat is guaranteed to be true by the information youhave been given. That is, the correct answer will be astatement that must be true if the given information istrue. Incorrect answer choices may receive some supportfrom the information but that support will be inconclu-sive. In other words, an incorrect answer choice could befalse even if the information provided is true.

An Example

Any sale item that is purchased can be returned for storecredit but not for a refund of the purchase price. Everyhome appliance and every piece of gardening equipment ison sale along with selected construction tools.8

The question asks:

If the statements above are true, which one of the followingmust also be true?

Notice that the statements have a common element: theytalk about sale items. This common element allows you tocombine bits of information to draw conclusions. For exam-ple, since all home appliances are sale items, you couldconclude that any home appliance that is purchased can be

A Guide to Logical Reasoning Questions20

8 December 2001 LSAT, section 1, question 11.

returned for store credit. Because several conclusions likethis can be drawn from these statements, you cannot deter-mine the correct answer without reading the answerchoices. So you need to go through the answer choices tofind one that must be true if the statements are true.

One choice reads:

No piece of gardening equipment is returnable for a refund.

We are told that every piece of gardening equipment isa sale item and sale items are not returnable for a refund.So it must be true that gardening equipment is notreturnable for a refund. This is the correct answer choice.

For the sake of comparison, consider anotheranswer choice:

Some construction tools are not returnable for store credit.

To rule out this answer choice, you need to see that itdoes not have to be true if the statements in the passageare true. It obviously doesn’t have to be true for construc-tion tools that are on sale—the statements guarantee thatthose construction tools are returnable for store credit. Asfor the rest of the construction tools, those that aren’t onsale, nothing indicates that they are not returnable forstore credit. Based on what the statements say, it is possi-ble, and even likely, that these tools are returnable forstore credit. The answer choice is therefore incorrect.

In this example, you were given a set of statements thatdo not seem to be designed to lead to any particular conclu-sion. It was up to you to determine the implications ofthose statements. In other cases, however, the informationmay appear to be designed to lead the reader to a specificunstated conclusion. In such cases, the correct answercould be the unstated conclusion, if it logically follows fromthe information provided, or it could be some other state-ment that logically follows from that information.

Some Points to Consider

� For some claim to logically follow from certain informa-tion, that information has to guarantee that the claim istrue. It isn’t enough for the information to strongly sup-port the claim; it has to conclusively establish the claim.

� Incorrect answers to questions about what logicallyfollows can be claims that receive some support fromthe information but that nevertheless could be falseeven though all of the information is correct.

� Answer choices are often incorrect because they takethings one step beyond what the evidence supports.They might make claims that are too sweeping; forexample, they might say “all” when the evidence sup-ports only a “most” statement. Or where a statementabout what “is likely to be” is warranted, an incorrectanswer choice might say “is.” Or where a statementabout “all known cases” is warranted, an incorrectanswer choice might say “all cases.”

� Remember that a modest or limited claim can be acorrect answer even if the information also supports astronger claim. If the information supports drawingthe conclusion that there will be a festival in everymonth, then it also supports the conclusion that therewill be a festival in June.

� Identifying a Position Supported byInformation Provided

Some questions ask you to identify a position that issupported by a body of evidence, but not supportedconclusively. These questions might be worded as follows:

Which one of the following is most strongly supported bythe information above?

Which one of the following can most reasonably be con-cluded on the basis of the information above?

The statements above, if true, most strongly supportwhich one of the following?

For these questions, you will generally not be pre-sented with an argument, but merely with some piecesof information. Your task is to evaluate that informationand distinguish between the answer choice that receivesstrong support from that information (the correctanswer) and answer choices that receive no significantsupport (the incorrect answer choices).

An Example

Consider the following pieces of information:

People should avoid taking the antacid calcium carbonate indoses larger than half a gram, for despite its capacity to neu-tralize stomach acids, calcium carbonate can increase thecalcium level in the blood and thus impair kidney function.Moreover, just half a gram of it can stimulate the productionof gastrin, a stomach hormone that triggers acid secretion.9

A Guide to Logical Reasoning Questions 21

9 June 2001 LSAT, section 2, question 19.

You are asked,

Which one of the following is most strongly supported bythe information above?

With questions of this kind you shouldn’t expect thecorrect answer to follow in a strict logical sense from theinformation, but you should expect the information toprovide a strong justification for the correct answer.When you begin work on a question of this sort, youshould note any obvious interconnections among thefacts given, but there is no point in formulating a preciseprediction of what the correct answer will look like. Asensible approach is to read the passage carefully, andmake a mental note of any implications that you spot.Then go on to consider each answer choice in turn anddetermine whether that answer choice gets any supportfrom the information you have been given.

Let’s follow this approach with the question above.Reading the passage, you find that a certain antacid isdescribed as having the obvious intended effect of neu-tralizing stomach acid but as also having adverse sideeffects if the dosage is too high. One of these adverseeffects results in impaired kidney function and otherresults in acid secretion in the stomach.

There is a suggestion in the passage that doses exceed-ing half a gram are necessary for the first effect to betriggered to any serious extent. The passage also sug-gests that doses of half a gram or more will trigger thesecond effect. No other implications of this passagestand out. At this point, it is probably a good idea to con-sider each answer choice in turn.

One answer choice is:

Doses of calcium carbonate smaller than half a gram canreduce stomach acid more effectively than much largerdoses do.

Is this choice supported by the information? The passagedoes give reasons as to why this might be true. It tells usthat doses of half a gram or more can stimulate the produc-tion of a stomach hormone that triggers acid secretion. Thishormone might counteract any extra acid-neutralizationthat comes from additional calcium carbonate over andabove a half-gram dose; but then again it might not. Per-haps the extra calcium carbonate neutralizes morestomach acid than it triggers. For this reason, this answerchoice is not strongly supported by the information.

Another answer choice is:

Half a gram of calcium carbonate can causally contribute toboth the secretion and the neutralization of stomach acids.

Is there support for this choice in the information pro-vided? We have noted that at half a gram the secretion ofacid in the stomach is triggered. The passage mentionsthe drug’s “capacity to neutralize stomach acids,”strongly suggesting that some acid-neutralizing effectoccurs at any dosage level. So there is strong support inthe passage for both parts of this answer choice.

Some Points to Consider

� In answering questions dealing with support for con-clusions, base your judgment about whether or not aparticular answer choice is supported strictly on theinformation that is explicitly provided in the passage.If the passage concerns a subject matter with whichyou are familiar, ignore any information you mighthave about the subject that goes beyond what youhave been told.

� Keep in mind that the support for the correct answerdoes not have to involve all of the information pro-vided. For instance, in the example about calciumcarbonate, an adverse effect on the kidneys is men-tioned, but this information plays no role in thesupport for the correct answer.

� Identifying Points on Which Disputants HoldConflicting Views

You may also encounter questions involving two speak-ers where the first speaker puts forward a position andthe second responds to that position. You will then beasked something like:

The main point at issue between Sarah and Paul is whether

Which one of the following most accurately expresses thepoint at issue between Juan and Michiko?

On the basis of their statements, Winchell and Trent arecommitted to disagreeing over whether

An Example

Mary: Computers will make more information available toordinary people than was ever available before, thus mak-ing it easier for them to acquire knowledge withoutconsulting experts.

A Guide to Logical Reasoning Questions22

Joyce: As more knowledge became available in previouscenturies, the need for specialists to synthesize andexplain it to nonspecialists increased. So computers willprobably create a greater dependency on experts.10

The question asks,

The dialogue most strongly supports the claim that Maryand Joyce disagree with each other about whether

In answering questions of this kind, you may find ituseful to read the dialogue closely enough to form aclear mental picture of each person’s stance and then goon to the answer choices.

Now consider this answer choice:

computers will make more information available toordinary people

Does what Joyce and Mary say show that they disagreeabout this? Mary straightforwardly says that computerswill make more information available to ordinary peo-ple. But what about Joyce? She predicts that computerswill create a greater dependency on experts because of ahistorical trend of an increasing dependency on expertswhenever more knowledge becomes available to ordi-nary people. So she seems to assume that computers willmake more information become available to ordinarypeople. So she probably agrees with Mary on this point.

Now consider a second answer choice:

dependency on computers will increase with the increaseof knowledge

Nothing either Mary or Joyce says commits either of themto a particular view on this position. This is because neitherof them explicitly discusses the issue of people’s depend-ency on computers. But there is certainly no indication atall that they hold opposing views on whether dependencyon computers will increase with the increase of knowledge.

Finally, consider a third answer choice:

computers will increase the need for ordinary people seek-ing knowledge to turn to experts

Based on what she says, Mary straightforwardly dis-agrees with this claim. Computers, she says, will make iteasier for ordinary people to acquire knowledge withoutconsulting experts. Joyce, on the other hand, concludesthat computers will create a greater dependency onexperts. The precedent from past centuries that she citesin support of this conclusion makes it clear that nonspe-cialists—that is, ordinary people—will depend more onexperts when knowledge increases. So Mary and Joycedisagree on whether the need for ordinary people to turnto experts will be increased by computers.

Some Points to Consider

� The evidence that two speakers disagree about aparticular point always comes from what they explic-itly say. Sometimes there is a direct conflict betweensomething that one of the speakers says and some-thing that the other speaker says. The phrasing of thequestion indicates that you should be looking for adirect conflict when it says something straightforwardlike “Max and Nina disagree over whether.” At othertimes the point of disagreement must be inferred fromthe explicit positions that the speakers take. Thephrasing of the question will indicate that this infer-ence needs to be made. For example, a question like“The dialogue provides the most support for theclaim that Nikisha and Helen disagree over whether”does not suggest that they disagree explicitly, onlythat there is some evidence that they disagree.

� Do not try to derive a speaker’s likely position on a topicfrom a psychological stereotype. It may be true that aspeaker who takes a certain position would be the kindof person who would likely hold certain other positionsas well, but you should not rely on this sort of associa-tion. Rely only on what a speaker explicitly says and onwhat can be properly inferred from that.

� The incorrect answer choices are not necessarily posi-tions that the two speakers can be shown to agree on.In many cases, the views of at least one of the speak-ers on a given position cannot be determined fromwhat has been said.

A Guide to Logical Reasoning Questions 23

10 December 2001 LSAT, section 3, question 4.

Necessary Conditions and Sufficient Conditions

Suppose you read the following statements:

You don’t deserve praise for something unless youdid it deliberately.

Tom deliberately left the door unlocked.

Does it follow from these statements that Tom deservespraise for leaving the door unlocked? You can probablysee that this doesn’t follow. The first statement says thatyou have to do something deliberately in order to deservepraise for doing it. It doesn’t say that any time you dosomething deliberately you thereby deserve praise fordoing it. So the mere fact that Tom did something deliber-ately is not enough to bring us to the conclusion that Tomdeserves praise for doing it.

To put it in a slightly more technical way, the firststatement expresses a necessary condition. Doing some-thing deliberately is a necessary condition for deservingpraise for doing it. In Logical Reasoning questions, it can bevery important to recognize whether something expresses anecessary condition or whether it expresses what is called asufficient condition. If the first statement had said “If youdo something deliberately then you deserve praise for doingit,” it would be saying that doing something deliberately isa sufficient condition for deserving praise for doing it.

Cross-reference: Reasoning involving necessaryand sufficient conditions is also covered, from aslightly different perspective, in the discussionof “Conditional Statements” in the AnalyticalReasoning section, pages 11–12.

In the example above, it is fairly easy to see that the firststatement expresses a necessary condition and not a suffi-cient condition. This may be because it would be quitestrange to say that doing something deliberately is a suffi-cient condition for deserving praise. That would implythat you deserve praise for anything you do deliberately,even if it is an immoral or criminal act. But the content ofa statement doesn’t always help you determine whether itexpresses a necessary condition or a sufficient condition.For this reason, it pays to devote very close attention tothe precise wording of any statements that express condi-tions. And it pays to have a clear idea in your mind abouthow statements that express necessary conditions func-tion in arguments and about how statements that expresssufficient conditions function in arguments.

There are many ways to express a necessary condition.The necessary condition above could have been statedjust as accurately in several different ways, including:

You deserve praise for something only if you did it deliberately.

You don’t deserve praise for something if you didn’t doit deliberately.

To deserve praise for something, you must have doneit deliberately.

If you think carefully about these statements, youshould see that they all mean the same thing. And youcan see that none of them says that doing somethingdeliberately is a sufficient condition for deserving praise.

Sufficient conditions can also be expressed in severaldifferent ways:

If it rains, the sidewalks get wet.

Rain is all it takes to get the sidewalks wet.

The sidewalks get wet whenever it rains.

These statements each tell us that rain is a sufficientcondition for the sidewalks getting wet. It is sufficient,because rain is all that it takes to make the sidewalkswet. But notice that these statements do not say that rainis the only thing that makes the sidewalks wet. They donot rule out the possibility that the sidewalks can get wetfrom melting snow or from being sprayed with a gardenhose. So these statements do not express necessary con-ditions for wet sidewalks, only sufficient conditions.

� How Necessary Conditions Work in Inferences

We’ve already noted one thing about basing inferenceson statements that express necessary conditions, such as

N: You deserve praise for something only if you didit deliberately.

If we are also given a case that satisfies the necessarycondition, such as

Tom deliberately left the door unlocked

we cannot legitimately draw an inference. Specifically,the conclusion that Tom deserves praise for leaving thedoor unlocked does not follow.

Statements that express necessary conditions can playa part in legitimate inferences, of course, but only incombination with the right sort of information. Supposethat in addition to statement N we are told

Tom deserves praise for leaving the door unlocked.

A Guide to Logical Reasoning Questions24

This allows us to conclude that Tom deliberately left thedoor unlocked. Since statement N says that you have to dosomething deliberately in order to deserve praise for doingit, Tom must have deliberately left the door unlocked ifhe deserves praise for what he did.

Or, suppose that in addition to statement N we are told

Tom did not leave the door unlocked deliberately.

This allows us to conclude that Tom does not deservepraise for leaving the door unlocked. This followsbecause statement N insists that only deliberate actionsdeserve praise, and because we are told clearly thatTom’s action is not deliberate.

So in general, when you have a statement that expresses anecessary condition, it allows you to infer something in justtwo cases: (1) you can infer from knowing that the neces-sary condition is not met that the thing it is the necessarycondition for does not occur; (2) you can infer that the nec-essary condition is met from knowing that the thing it is thenecessary condition for occurs.

� How Sufficient Conditions Work in Inferences

Statements that express sufficient conditions can alsoserve as a basis for inferences. Let’s revisit one of theearlier statements of a sufficient condition:

S: If it rains, the sidewalks get wet.

If we are told that the sufficient condition is satisfied(i.e., told that it is raining), then we can legitimatelydraw the inference that the sidewalks are getting wet.This should be quite obvious.

We can also draw another conclusion from a statementof a sufficient condition, provided that we have theright sort of additional information. Suppose that in addi-tion to statement S we are told that the sidewalks did notget wet. Since the sidewalks get wet whenever it rains, wecan conclude with complete confidence that it didn’t rain.

So in general, when you have a statement that expressesa sufficient condition, it allows you to infer something injust two cases: (1) if you know that the sufficient conditionis met, then you can infer that the thing it is the sufficientcondition for occurs; (2) you can infer that the sufficientcondition is not met from knowing that the thing it is thesufficient condition for does not occur.

Though it may sometimes seem that there are otherways to draw an inference from a statement of a suffi-cient condition, there are none. Suppose that in additionto statement S, we are told that the sidewalks are wet.Can we legitimately conclude that it rained? No, becausestatement S does not rule out the possibility that some-thing other than rain, such as melting snow, can makethe sidewalks wet. Or suppose that in addition to state-ment S, we are told that it didn’t rain. Can welegitimately conclude that the sidewalks did not get wet?Again no, and for the same reason: statement S does notrule out the possibility that something other than raincan make the sidewalks wet.

Understanding the Impact of Additional Information

The LSAT typically includes several questions that testyour ability to see how additional facts bear on an argument.These questions may focus on facts that strengthen anargument, they may focus on facts that weaken the argument,or they may merely ask what additional information, if itwere available, would be useful in evaluating the strengthof the argument. Typical wordings of such questions are:

Which one of the following, if true, most strengthensthe argument?

Which one of the following, if true, most weakens the argument?

In order to evaluate the argument, which one of thefollowing would it be most useful to determine?

Tip: When the qualifier “if true” appears in thiskind of question, it tells you not to be concernedabout the actual truth of the answer choices.Instead, you should consider each answerchoice as though it were true. Also, considereach answer choice independently of the others,since it is not necessarily the case that theanswer choices can all be true together.

Questions of this kind are based on argumentsthat—like most real-life arguments—have premises thatprovide some grounds for accepting the conclusion, butthat fall short of being decisive arguments in favor of theconclusion. For an argument like this, it is possible foradditional evidence to make the argument stronger orweaker. For example, consider the following argument:

A survey of oil-refinery workers who work with MBTE, aningredient currently used in some smog-reducing gaso-lines, found an alarming incidence of complaints aboutheadaches, fatigue, and shortness of breath. Since gasoline

A Guide to Logical Reasoning Questions 25

containing MBTE will soon be widely used, we can expectan increased incidence of headaches, fatigue, and shortnessof breath.11

The incidence of complaints about headaches, fatigue,and shortness of breath among oil-refinery employeeswho work with MBTE is, on the face of it, evidence forthe conclusion that widespread use of gasoline contain-ing MBTE will make headaches, fatigue, and shortnessof breath more common. However, additional informa-tion could, depending on what this information is, makethe argument stronger or weaker.

For example, suppose it is true that most oil-refineryworkers who do not work with MBTE also have a veryhigh incidence of headaches, fatigue, and shortness ofbreath. This would provide evidence that it is not MBTEbut some other factor that is primarily responsible forthese symptoms. But if we have evidence that somethingother than MBTE is causing these symptoms, then theargument provides only very weak support, if any, for itsconclusion. That is, the argument’s original premises,when combined with the additional information, make amuch weaker case for the argument’s conclusion thanthose premises did alone. In other words, the new infor-mation has made the argument weaker.

Of course, different additional evidence would make theargument stronger. For example, suppose that gasolinecontaining MBTE has already been introduced in a fewmetropolitan areas, and since it was first introduced,those areas have reported increased complaints aboutheadaches, fatigue, and shortness of breath. This wouldprovide evidence that when MBTE is used as a gasolineadditive, it increases the incidence of these symptomsnot just among refinery workers who work closely withit but also among the general public. So we now haveevidence that is more directly relevant to the argument’sconclusion. Thus, we now have a stronger case for theargument’s conclusion; in other words, the new evidencehas made the argument stronger.

We have seen that when new information makes anargument stronger, that information, together with theargument’s original premises, makes a stronger case forthe argument’s conclusion than do the original premisesalone. There are several ways in which this could work.The additional information could eliminate an obviousweak spot in the original argument. Alternatively, theremay be no obvious weak spot in the original argument;the case for the argument may simply become evenstronger with the addition of the new evidence. In somecases, the additional information will be something thathelps establish the argument’s conclusion but only when

combined with the argument’s existing premises. Inother cases, the new information will provide a differentline of reasoning in addition to that provided by theoriginal premises. The information that strengthens theargument about MBTE is an example of something thatprovides a different line of reasoning for the conclusion.In still other cases, the additional information willstrengthen the argument by ruling out something thatwould have weakened the argument. And of course,additional information may weaken an argument in cor-responding ways.

An Example

Consider this argument:

A recent study reveals that television advertising does notsignificantly affect children’s preferences for breakfast cere-als. The study compared two groups of children. Onegroup had watched no television, and the other group hadwatched average amounts of television and its advertising.Both groups strongly preferred the sugary cereals heavilyadvertised on television.12

The conclusion of the argument is that televisionadvertising does not significantly affect children’s pref-erences for breakfast cereals. As evidence for thisconclusion, the argument presents the results of a studycomparing two groups of children: the study found thatchildren in both groups—those who watched no televisionand those who watched average amounts of television andits advertising—strongly preferred the sugary cerealsheavily advertised on television. On the face of it, the studyresults do seem to provide some support, although notconclusive support, for the argument’s conclusion; iftelevision advertising did significantly affect children’spreferences, then we’d expect the children who watchedtelevision to have different preferences than the childrenwho didn’t watch television.

Here is the question:

Which one of the following statements, if true, most weak-ens the argument?

Let’s consider an answer choice:

Most of the children in the group that had watchedtelevision were already familiar with the advertisements forthese cereals.

A Guide to Logical Reasoning Questions26

11 October 2002 LSAT, section 4, question 5.12 October 2002 LSAT, section 4, question 13.

Does this information weaken the argument? It sug-gests that even if the television advertising influencedthe preferences of the children who watched television,this influence occurred some time ago. But this does notreally imply anything about whether the advertising didinfluence the children’s preferences. So the informationprovided by this answer choice neither strengthens norweakens the argument.

Let’s consider another answer choice:

Both groups rejected cereals low in sugar even when thesecereals were heavily advertised on television.

This information may well be relevant to the argu-ment’s conclusion since it suggests that if a cereal isunappealing to children, then even a great deal oftelevision advertising will not change the children’spreferences. But this would provide additional evidencein favor of the argument’s conclusion that televisionadvertising does not significantly affect children’s cerealpreferences. So this answer choice strengthens the argu-ment rather than weakens it.

Tip: In questions that ask what weakens anargument, often one or more incorrect answerchoices will provide evidence that strengthensthe argument (or vice versa in the case ofquestions that ask for a strengthener). By thetime you’ve read several answer choices, it iseasy to forget what the question is asking forand pick an answer choice because it is clearlyrelevant—even though it’s the opposite of whatthe question is asking for. It is important tokeep the question clearly in mind in order toguard against making this kind of mistake.

Consider a third answer choice, then:

The preferences of children who do not watch televisionadvertising are influenced by the preferences of childrenwho watch the advertising.

How does this information affect the argument? Well, thereason originally offered for the conclusion is that thetwo groups of children do not differ in their preferences.But if the preferences of the children who do not watchtelevision advertising are influenced by the preferencesof those who do watch it, then the fact that the twogroups do not differ in their preferences provides little, ifany, reason to think that none of the children’spreferences were affected by television advertising. Afterall, it could well be that the preferences of the childrenwho watched television were strongly influenced by theadvertising, and these children’s preferences in turn

strongly influenced the preferences of those who did notwatch television, with the result that the two groups hadthe same preferences. So when combined with the addi-tional information, the argument’s original premisesmake a much weaker case for the argument’s conclusionthan they did alone. Thus, this is the correct answer.

More Points to Consider

� The additional pieces of information that weaken anargument generally do not challenge the truth of theargument’s explicit premises. They are pieces of infor-mation that call into question whether the conclusionis strongly supported by those premises.

� Keep in mind that additional information maystrengthen (or weaken) an argument only to a smallextent or it may do so to a large extent. When thequestion asks for a strengthener, an answer choicewill be correct even if it strengthens the argumentonly slightly, provided that none of the other answerchoices strengthen the argument significantly. On theother hand, if one answer choice strengthens theargument a great deal, then answer choices thatstrengthen only slightly are incorrect. For most ques-tions that ask for weakeners, the correct answer willweaken the argument to some extent, but the pre-mises will still provide some support for theconclusion. However, for some of these questions, thecorrect answer will eliminate all or almost all of theargument’s original strength.

� Beware of answer choices that are relevant to the gen-eral subject matter, but not relevant to the way theargument supports its conclusion. A weakener orstrengthener must affect the support for the conclu-sion. For example, consider the argument about gasolinecontaining MBTE. Suppose that adding MBTE to gaso-line dramatically increased the price of gasoline. Thisinformation would be relevant if the argument’s conclu-sion were broader, for example, if it concluded thatgasoline containing MBTE should be widely used. Sincethe argument, however, is narrowly focused on whetherwidespread use of gasoline containing MBTE willincrease the incidence of headaches, fatigue, andshortness of breath, the increased cost resulting fromadding MBTE to gasoline is irrelevant and thus wouldneither strengthen nor weaken the argument.

� Similarly, for new information to weaken an argu-ment, it must reduce the support that the premisesprovide for the conclusion. A fact may have negativeconnotations in the context of an argument but donothing to weaken that argument. For example, con-sider the argument about television advertising andcereal preferences. Suppose that children who watchaverage amounts of television, unlike children who

A Guide to Logical Reasoning Questions 27

watch no television, do not get enough exercise. Thiswould clearly be a negative aspect of watching televi-sion. But it doesn’t weaken the support that theargument provides for the conclusion that television

advertising does not significantly affect children’spreferences for breakfast cereal.

Assumptions

The Logical Reasoning section typically includes severalquestions that test your ability to identify assumptionsof arguments. An assumption of an argument plays arole in establishing the conclusion. However, unlike apremise, an assumption is not something that the arguerexplicitly asserts to be true; an assumption is instead justtreated as true for the purposes of the argument.

Although assumptions can be stated explicitly in anargument, Logical Reasoning questions that ask aboutassumptions ask only about unstated assumptions.Unstated (or tacit) assumptions can figure only inarguments that are not entirely complete, that is, in argu-ments in which some of the things required to establishthe conclusion are left unstated. There is thus at least onesignificant gap in such an argument.

Assumptions relate to the gaps in an argument in twodifferent ways. An assumption is a sufficient one ifadding it to the argument’s premises would produce aconclusive argument, that is, an argument with no gapsin its support for the conclusion. An assumption is anecessary one if it is something that must be true inorder for the argument to succeed. It is possible for anassumption to be both necessary and sufficient.

� Sufficient Assumptions

Typical wordings of questions that ask you to identifysufficient assumptions are:

Which one of the following, if assumed, enables the conclu-sion of the argument to be properly drawn?

The conclusion follows logically from the premises if whichone of the following is assumed?

An Example

Vague laws set vague limits on people’s freedom, whichmakes it impossible for them to know for certain whether theiractions are legal. Thus, under vague laws people cannotfeel secure.13

The question you’re asked about this argument is:

The conclusion follows logically if which one of the follow-ing is assumed?

In order to approach this question, you first have toidentify the conclusion of the argument and the premisesoffered in its support. In this case, the conclusion is sig-naled by the conclusion indicator “thus” and reads“…under vague laws people cannot feel secure.” Twoconsiderations are explicitly presented in support of thisconclusion. First, that vague laws set vague limits onpeople’s freedom, and second, that having vague limitsset on their freedom makes it impossible for people toknow for certain whether their actions are legal. Notethat the premises, though they tell us certain things aboutvague laws, make no explicit reference to whether peoplefeel secure, and not feeling secure is what the conclusion isabout. For the conclusion to follow logically, this gap has tobe bridged.

At this point, you are ready to look at the answerchoices. Here are two of them:

(A) People can feel secure only if they know for certainwhether their actions are legal.

(B) If people know for certain whether their actions arelegal, they can feel secure.

Your task is to identify the answer choice that, togetherwith the premises you’ve been given, will provide con-clusive support for the conclusion.

So is (A) that answer choice? The explicit premises ofthe argument tell you that under vague laws people can-not know for certain whether their actions are legal. (A)tells you that if people do not know for certain whethertheir actions are legal, they cannot feel secure. So puttingthe explicit premises and (A) together, you can infer thatunder vague laws people cannot feel secure. And this is,in fact, the conclusion of the argument. So the conclusionfollows logically if (A) is assumed.

Now, let’s consider why assuming (B) is not sufficientto ensure that the argument’s conclusion followslogically. (B) tells us about one circumstance in whichpeople can feel secure. However, the argument’s conclu-sion will not follow logically without the right kind ofinformation about the circumstances in which people

A Guide to Logical Reasoning Questions28

13 December 2001 LSAT, section 3, question 12.

cannot feel secure. (B) does not give us any such infor-mation directly. Moreover, we cannot infer suchinformation from what (B) does tell us. After all, it’s per-fectly compatible with (B) that people can feel secure insome circumstances in addition to the one (B) describes.For example, perhaps people can feel secure if theyknow for certain that they will not be prosecuted fortheir actions. Thus, since (B) tells us nothing about cir-cumstances in which people cannot feel secure, it hasnothing to contribute to reaching the argument’s conclu-sion that people cannot feel secure under vague laws.

Some Points to Consider

� In answering sufficient assumption questions, you needto find a link between the stated premises and the con-clusion. Try to determine from the explicit parts of theargument what logical work that link needs to do.Finally, look among the answer choices for one thatcan do that logical work and that, taken along withthe explicit premises, allows the conclusion to beproperly inferred.

� In trying to figure out what logical work the linkneeds to do, don’t get too specific. For example, whatcan be said about the logical work required of the linkin the argument about vague laws analyzed above? Ithas to link something that has been explicitly con-nected with vague laws to an inability to feel secure.But there are two things like that: vague limits onpeople’s freedom, and the impossibility of knowingfor certain whether one’s actions are legal. What thismeans is that answer choice (A) was not the onlypossible sufficient assumption here. An equallyacceptable sufficient assumption would have been,“People cannot feel secure if they have vague limitson their freedom.” So don’t approach the answerchoices with too specific a view of what you’relooking for.

� When trying to identify a sufficient assumption, keepin mind that the correct answer must, when added tothe argument’s explicit premises, result in a conclu-sive argument; that is, in an argument that fullyestablishes its conclusion (provided that the explicitpremises and the added assumption are all true).

� Necessary Assumptions

Typical wordings of questions that ask you to identifynecessary assumptions include the following:

The argument relies on assuming which one ofthe following?

The argument depends on the assumption that

Which one of the following is an assumption required bythe argument?

Questions about necessary assumptions refer to argu-ments that, while not completely spelled out, do present acomprehensible case for accepting their conclusion on thestrength of evidence explicitly presented. But if you lookclosely at the grounds offered for the conclusion and atthe conclusion itself, you find that the evidence explicitlypresented falls short of establishing the conclusion. Thatis, there is at least one significant gap in the argument.

Example 1

Since Mayor Drabble always repays her political debts assoon as possible, she will almost certainly appoint Lee to bethe new head of the arts commission. Lee has wanted thatjob for a long time, and Drabble owes Lee a lot for his sup-port in the last election.14

As far as its explicit premises go, this argument leavesimportant matters unresolved. In order for the argument toshow that Lee is the likely appointee, there can’t be anyoneelse to whom Drabble has owed a such a large andlong-standing political debt and for whom this appoint-ment would be adequate repayment. This idea of therebeing no one ahead of Lee in line is the sort of unstatedbut indispensable link in the support for the conclusionthat we mean when we speak of a necessary assumptionof an argument.

It can readily be shown that the assumption sketchedabove is in fact indispensable to the argument. Supposethe situation were otherwise and there were a person towhom Mayor Drabble owed a political debt that is oflonger standing than her debt to Lee, and suppose fur-ther that the appointment could reasonably be viewed aspaying off that debt. In this hypothetical circumstance,the fact that Mayor Drabble always repays her politicaldebts as soon as possible would no longer point to Leeas the likely choice for the appointment. In fact, theargument above would fail. If the argument is tosucceed, there cannot be another, better-positioned can-didate for the appointment. And the argument dependson the assumption that there isn’t any better-positioned candidate.

A Guide to Logical Reasoning Questions 29

14 June 2004 LSAT, section 2, question 24.

A Test for Necessary Assumptions

Necessary assumption questions, then, require you toidentify tacit assumptions. The method for testing neces-sary assumptions that was introduced above in analyzingMayor Drabble’s situation is quite generally applicable,and for good reason. A necessary assumption is an indis-pensable link in the support for the conclusion of anargument. Therefore, an argument will be ineffective if anecessary assumption is deemed to be false. This points toa useful test: to see whether an answer choice is a necessaryassumption, suppose that what is stated in that answerchoice is false. If under those circumstances the premisesof the argument fail to support the conclusion, the answerchoice being evaluated is a necessary assumption.

Example 2

The test for necessary assumptions can be used withthe following argument:

Advertisement: Attention pond owners! Ninety-eight per-cent of mosquito larvae in a pond die within minutes afterthe pond has been treated with BTI. Yet BTI is not toxic tofish, birds, animals, plants, or beneficial insects. So byusing BTI regularly to destroy their larvae, you cangreatly reduce populations of pesky mosquitoes that hatchin your pond, and can do so without diminishing the pop-ulations of fish, frogs, or beneficial insects in and aroundthe pond.15

The question asks:

Which one of the following is an assumption on which theargument depends?

Before you look for a necessary assumption, you needto get clear about the structure of the argument. The con-clusion is that regular applications of BTI in a pond can,without reducing populations of assorted pond life,greatly reduce the numbers of mosquitoes that emergefrom the pond. The evidence is that BTI kills almost allof the mosquito larvae in the pond, but does not kill (oreven harm) other pond life.

The case that the argument makes for its conclusion isstraightforward. Applications of BTI, by killing mos-quito larvae, prevent the adult mosquito populationfrom being replenished, but they have no direct effect onthe other populations. So the argument concludes that,of the populations under consideration, only the mos-quito populations will decline.

The first answer choice reads:

The most effective way to control the numbers of mosquitoesin a given area is to destroy the mosquito larvae inthat area.

Now we apply the test for necessary assumptions byasking whether the argument would fail if this answerchoice were false. That is, would it fail if the destruction ofmosquito larvae were not the most effective way to con-trol the numbers of mosquitoes? Definitely not. For onething, the argument is not concerned with mosquito con-trol alone, but speaks to a dual purpose, that ofcontrolling mosquitoes while leaving other creaturesunaffected. So the potential existence of any mos-quito-control regimen, however effective, that did notspare other pond creatures would be beside the point. Foranother thing, the argument merely concludes that the useof BTI works, not that it works better than all other meth-ods. So the denial of this answer choice does not interferewith the support that the conclusion receives from the evi-dence presented. But if this answer choice were a necessaryassumption, denying it would interfere with that support.

Now consider a second answer choice:

The fish, frogs, and beneficial insects in and around apond-owner’s pond do not depend on mosquito larvae asan important source of food.

Applying the test, we ask whether the argumentwould fail if this answer choice were false (that is, ifthese creatures did depend on mosquito larvae for food).Yes it would; after all, if the use of BTI means that fish,frogs, and so forth will be deprived of a food that is impor-tant for them (mosquito larvae), then there is no reason toconclude that these creatures will survive in undiminishednumbers. So denying the answer choice under consider-ation would cause the argument to fail; we have found anecessary assumption.

Some Points to Consider

� As you can see from the characterization of necessaryassumptions given above, they are (unstated) constit-uents of arguments. Whether or not the author of theargument had a particular assumption in mind is notrelevant to the issue. It is important to remember thatidentifying necessary assumptions is a matter oflogically analyzing the structure of an argument, nota matter of guessing the beliefs of the arguer.

A Guide to Logical Reasoning Questions30

15 February 1994 LSAT, section 4, question 18.

� For the purpose of identifying a necessary assumption,it is not necessary or even useful to evaluate whetherthat assumption is actually true, or how likely it is tobe true. Identifying an assumption is a matter of prob-ing the structure of an argument and recognizinghidden parts of that structure.

� An argument may have more than one necessaryassumption. For example, the argument in Example 2ignores the fact that a small proportion of mosquitolarvae in a pond are not killed by BTI. But if there is agenetic basis for their not being killed, one might imag-ine that regular applications of BTI in a given pond willmake it more and more likely that the mosquitoes left tobreed with one another will be BTI-resistant ones thatwill likely produce BTI-resistant offspring. This popula-tion of BTI-resistant mosquitoes might then grow,without being kept in check by further applications ofBTI, contrary to the drift of the argument. So the argu-ment also depends on assuming that the two percent ofmosquito larvae not killed by an initial application ofBTI do not constitute an initial breeding pool for aBTI-resistant population of mosquitoes.

An argument can thus have more than one neces-sary assumption. Of course, only one of them will

appear among the answer choices. But the one thatdoes appear may not be one that occurred to youwhen you analyzed the argument. So it is a good ideanot to prejudge what the correct answer will be.Instead, keep an open mind and examine each of theanswer choices in turn.

� As indicated above, an argument may have more thanone gap. Any one necessary assumption will addressonly one such gap. Moreover, a necessary assumptionwill often address only some aspects of a gap. InExample 2, the gap addressed by the necessaryassumption—The fish, frogs, and beneficial insects in andaround a pond-owner’s pond do not depend on mosquitolarvae as an important source of food—is, broadlyspeaking, that BTI does not kill the fish, frogs, andbeneficial insects indirectly. But food deprivation isnot the only way that BTI might kill those creaturesindirectly. For example, as the mosquito larvae killedby applications of BTI decay, they might harm fish,frogs, and beneficial insects.

So do not reject an answer choice as a necessaryassumption merely on the grounds that the argument,even if you make that necessary assumption, is stillnot a strong argument.

Principles

Some Logical Reasoning questions test your ability toapply general rules and principles and to understandtheir use. These questions can involve the use of princi-ples in arguments, or they can involve applyingprinciples to actions or states of affairs.

Principles are broad guidelines concerning what kinds ofactions, judgments, policies, and so on are appropriate.Most principles spell out the range of situations to whichthey apply. Within that range of situations, principles oftenserve to justify the transition from claims about what is thecase to conclusions regarding what should be done.

There are several kinds of questions involving princi-ples. You may be given a principle and be asked whichaction conforms to it, or which judgment it justifies, orwhich argument relies on it. Alternatively, the questionmay present a judgment, decision, or argument and askwhich principle is appealed to in making that judgment,decision, or argument. Logical Reasoning questions mayalso involve principles in various other ways. For example,a question could ask which action violates a principle. Youmay also see Logical Reasoning items that ask you to recog-nize two situations as involving the same underlyingprinciple, where that principle is not stated.

� Applying a Principle That Is Given

An Example

People who receive unsolicited advice from someone whoseadvantage would be served if that advice is taken shouldregard the proffered advice with skepticism unless there isgood reason to think that their interests substantiallycoincide with those of the advice giver in the circumstancein question.16

The following question refers to this principle:

This principle, if accepted, would justify which one of thefollowing judgments?

The correct answer is provided by the judgment that ispresented below.

While shopping for a refrigerator, Ramón is approached bya salesperson who, on the basis of her personal experience,warns him against the least expensive model. However,the salesperson’s commission increases with the price of

A Guide to Logical Reasoning Questions 31

16 December 1992 LSAT, section 4, question 19.

the refrigerator sold, so Ramón should not reject the leastexpensive model on the salesperson’s advice alone.

The task here is to check how well the particulars ofthe situation fit with the principle. Do the general termsin which the principle is expressed cover the specific cir-cumstances of the situation?

So first you should ask, “Does someone in this situa-tion receive unsolicited advice from someone whoseadvantage would be served if that advice is taken?” Ifthe answer is “yes,” then the case under considerationfalls within the range of situations to which the principleapplies. If the answer is “no,” then the principle offersno guidance. In this situation, someone—Ramón—doesreceive advice. If Ramón took the advice, this would beto the advantage of the advice giver (the salesperson),because the salesperson would receive a highercommission than she would otherwise. Is the adviceunsolicited? Yes, because the salesperson approachedRamón without his asking for help.

The next question you should ask is, “Does the situa-tion culminate in a judgment that the advice should beregarded with skepticism?” The answer is again “yes.”The judgment that Ramón should not reject the leastexpensive model solely on the salesperson’s advice is ajudgment that treats the advice given—to avoid buyingthe least expensive model—skeptically.

You are not quite done at this point. The principlerestricts itself to situations in which the person givingthe advice and the person receiving the advice do nothave interests that coincide. So you need to ask one morequestion: “Is there reason to think that the interests ofRamón and those of the salesperson substantially coin-cide in this matter?” Since Ramón probably wants tospend no more than he has to and since the salespersonprobably wants Ramón to spend freely, there is reason tothink that in this matter their interests do not coincide.So the principle applies to the situation and justifiesthe judgment.

� Identifying a Guiding Principle

In the example above, the passage contained a principleand you were asked which judgment it justified. Thereare also questions that present a judgment or argumentin the passage and ask which principle justifies it.

An Example

Marianne is a professional chess player who hums audiblywhile playing her matches, thereby distracting her oppo-nents. When ordered by chess officials to cease humming

or else be disqualified from professional chess, Marianneprotested the order. She argued that since she was unawareof her humming, her humming was involuntary and thattherefore she should not be held responsible for it.17

The question that is based on this passage is:

Which one of the following principles, if valid, most helpsto support Marianne’s argument against the order?

To answer this question, you need to compare the spe-cific circumstances presented in the passage with theprinciple presented in each answer choice. First, considerthe following answer choice:

Of a player’s actions, only those that are voluntary shouldbe used as justification for disqualifying that player fromprofessional chess.

Does this principle apply to Marianne’s situation?Well, it is clear that the principle concerns which of a chessplayer’s actions can appropriately be used as justification fordisqualifying that player from professional chess. Since theargument in the passage is concerned with whether one ofMarianne’s actions—humming while playing—shoulddisqualify her from professional chess, it definitely fallsunder the range of situations to which the principle applies.

The principle will help support Marianne’s argument ifit leads to a judgment that Marianne’s humming whileplaying should not be used as justification for disquali-fying her from playing. According to a subsidiaryconclusion of Marianne’s argument, her humming isinvoluntary (this is supported by the claim that she wasunaware of it). The principle asserts that only voluntaryactions should be used as justification for disqualifying aplayer from professional chess, so this principle, togetherwith the subsidiary conclusion of Marianne’s argument,leads to the judgment that Marianne’s humming shouldnot be used as justification for disqualifying her. Thusthe principle does help support Marianne’s argument.

For the sake of comparison, consider one of the otheranswer choices:

Chess players should be disqualified from professional chessmatches if they regularly attempt to distract their opponents.

Does this principle apply to Marianne’s situation? Yes, itapparently does since it is also about the conditions underwhich chess players should be disqualified from profes-sional chess matches. So now you need to ask, does theprinciple establish, or help establish the conclusion that

A Guide to Logical Reasoning Questions32

17 October 1996 LSAT, section 4, question 24.

Marianne should not be disqualified for humming duringmatches? The answer is no. This principle just gives onecondition under which a chess player should be disquali-fied—when that player regularly attempts to distractopponents. Since Marianne’s humming is, she argues,involuntary, we can conclude that she is not trying to dis-tract her opponents. Thus the principle does not lead to thejudgment that Marianne should be disqualified for hum-ming. But this does not mean that the principle leads to thejudgment that Marianne should not be disqualified. Afterall, it is compatible with the principle that there are otherconditions under which a player should be disqualified,and such conditions could include humming while play-ing. So the principle does not lead to the conclusion thatMarianne should not be disqualified from professionalmatches and thus does not provide any support forMarianne’s argument.

Cross-reference: Making a common reasoningerror known as confusion between “sufficientconditions” and “necessary conditions” couldlead one to pick this answer choice. There aremany opportunities for this confusion to arisein principles questions. Further discussion ofsufficient and necessary conditions, and ofcorrect and incorrect ways of reasoning withconditions, can be found on pages 24–25.

Note that the process of finding the correct answer to aquestion that asks you to identify a guiding principle isbasically the same as it is for a question that asks you toapply a given principle. The task is to make sure that aprinciple, which will be couched in general terms, fitsthe particulars of the situation. The only difference isthat in the one case you need to check five different situ-ations against one principle, and in the other case youneed to check a single situation against five different

principles. But the way you determine the best fit is thesame. You check the principle for applicability, and youcheck to see whether the judgment, decision, recommen-dation, or action is in line with what the principle says.

Some Points to Consider

� Although principles always go beyond the particularcase, they can vary enormously in degree of general-ity and abstractness. For example, the principle in thequestion about Ramón’s refrigerator shopping is verygeneral, applying to anyone who receives unsolicitedadvice. In contrast, the principle in the question aboutMarianne the chess player is more specific, applyingonly to professional chess players. And some ques-tions have principles that can be even more specificthan this. So be sure not to reject a principle as a cor-rect answer solely because it seems to be too specific.

� When answering questions involving principles, it isalways a good idea to check all of the answer choices.There is no sure way to determine whether a givenanswer choice provides the best fit except by consid-ering and comparing all of the answer choices.

� Don’t reject a principle as providing justification foran argument merely because it seems to do no morethan spell out what the argument takes for granted.For example, someone might say, “Gerry sees verypoorly without glasses. Therefore, Gerry shouldalways wear glasses when driving.” A principle justi-fying this judgment might be “Anyone who needsglasses to see well should always wear glasseswhen driving.”

� Don’t worry about the legitimacy of any of the princi-ples you are presented with. Do not reject an answerbecause the principle involved is one that you person-ally would not accept.

Flaws in Arguments

� Identifying Argument Flaws

The Logical Reasoning section includes a number of ques-tions that ask you to identify a flaw of reasoning that hasbeen committed in an argument. Questions of this kind areworded in a variety of ways. Here are some examples:

The reasoning in the argument is flawed becausethe argument

The argument commits which one of the following errorsof reasoning?

The argument’s reasoning is questionable because theargument fails to rule out the possibility that

The reasoning above is most vulnerable to criticism on thegrounds that it

Test questions about flawed reasoning require you torecognize in what way an argument is defective in its rea-soning. They will not require you to decide whether or notthe argument is flawed. That judgment is already madeand is expressed in the wording of the question. Your taskis to recognize which one of the answer choices describesan error of reasoning that the argument makes.

A Guide to Logical Reasoning Questions 33

When an argument is flawed, the argument exempli-fies poor reasoning. This is reasoning in which thepremises may appear to provide support for the conclu-sion but actually provide little or no real support. Poorreasoning of this sort can be detected by examining theargument itself, without considering any factual issuesthat aren’t mentioned in the argument.

Tip: Logical Reasoning questions in the LSATtest your skills in reasoning and analysis, notwhat you know about any particular subjectmatter. Whether a premise is factually accurateis not relevant. So don’t pay attention to thefactual accuracy of an argument’s premises.Focus instead on logical connections betweenthe premises that the argument sets out andthe conclusion that it draws.

Since the flaws that you’ll be looking for arenot specific to the subject matter but relate tothe argument’s logical structure, the character-ization of those flaws can be quite general. Thatis to say, the flawed reasoning in an argumentmight be described in terms that also apply toother arguments that commit the same error ofreasoning. So once you detect where a particu-lar argument has gone wrong, you may thenhave to figure out which of several quite gen-eral descriptions covers the case.

Example 1

Consider the following brief exchange:

Physicist: The claim that low-temperature nuclear fusioncan be achieved entirely by chemical means is based onchemical experiments in which the measurements and cal-culations are inaccurate.

Chemist: But your challenge is ineffectual, since you are sim-ply jealous at the thought that chemists might have solved aproblem that physicists have been unable to solve.18

Here is the question that is based on this exchange:

Which one of the following is the strongest criticism of thechemist’s response to the physicist’s challenge?

Before looking at the answer choices, briefly considerwhat appears to be wrong with the chemist’s response.Notice that the chemist claims that the physicist’s challenge

is ineffectual but doesn’t actually engage the substance ofthe physicist’s challenge. Instead, the chemist accuses thephysicist of professional jealousy and dismisses the physi-cist’s challenge purely on that basis. But there is no reasonto think that a challenge, even if it is fueled by jealousy,cannot be on target. So the chemist’s response can rightlybe criticized for “getting personal.”

Now consider two of the answer choices. One ofthem reads,

It fails to establish that perfect accuracy of measurementsand calculations is possible.

This statement is certainly true about the chemist’sresponse. The chemist does not establish that perfectaccuracy is possible. But this is not a good criticism ofthe chemist’s response because it is entirely beside thepoint. Establishing that perfect accuracy is possiblewould have, if anything, damaged the chemist’s posi-tion. So the chemist’s response cannot be legitimatelycriticized for failing to establish this.

Another answer choice reads,

It is directed against the proponent of a claim rather thanagainst the claim itself.

This criticism goes to the heart of what is wrong withthe chemist’s response. The chemist dismisses the physi-cist’s challenge because of the physicist’s allegedmotives for making it and never actually discusses themerits of the challenge itself. It is directed against theperson rather than against the position.

In this example, the chemist’s response is clearlyirrelevant to the substance of the physicist’s claim. Theargument that the chemist presents seems more like a rhe-torical ploy than a serious argument. Many arguments areflawed in much less dramatic ways, however. They maycontain only a small logical lapse that undermines theintegrity of the argument, like the following two examples.

Example 2

Morris High School has introduced a policy designed toimprove the working conditions of its new teachers. As aresult of this policy, only one-quarter of all part-timeteachers now quit during their first year. However, athird of all full-time teachers now quit during their firstyear. Thus, more full-time than part-time teachers atMorris now quit during their first year.19

A Guide to Logical Reasoning Questions34

18 June 1995 LSAT, section 2, question 2.19 October 1994 LSAT, section 2, question 14.

Notice that the argument uses proportions to indicatethe degree to which first-year teachers are quitting. Itsays that one-quarter of part-time first-year teachers quitand that one-third of full-time first-year teachers quit.The conclusion of the argument is not expressed in termsof proportions, however, but in terms of a comparisonbetween quantities: more full-timers than part-timersquit during their first year.

Your task is to accurately complete the following statement:

The argument’s reasoning is questionable because theargument fails to rule out the possibility that

Note that we are looking for a possibility that needs tobe ruled out in order for the conclusion to be well sup-ported. So let’s consider one of the answer choices:

before the new policy was instituted, more part-time thanfull-time teachers at Morris High School used to quit dur-ing their first year

How would the argument be affected by this informa-tion? It tells us something about the way things werebefore the new policy went into effect, but it doesn’tshed much light on the effects of the new policy. And thereis no way to infer anything about how many part-time andfull-time teachers are quitting now, after the policy wasinstituted. So this information has no effect on the supportfor the conclusion, and there would be no reason for theargument to rule it out. Failing to rule it out, then, wouldnot make the reasoning questionable.

Let’s go on to consider another answer choice:

Morris High School employs more new part-time teachersthan new full-time teachers

So how would the argument be affected if there weremore new part-time teachers than new full time teachers?If there were more new part-timers than full-timers, thenone-quarter of the new part-timers could outnumberone-third of the new full-timers. So it could be true thatmore part-timers than full-timers quit during their firstyear. Since the argument concludes that more full-timersthan part-timers quit in their first year, this possibilityneeds to be ruled out in order for the conclusion to bewell supported. Thus, this choice is the correct answer.

Example 3

If Blankenship Enterprises has to switch suppliers in themiddle of a large production run, the company will not

show a profit for the year. Therefore, if Blankenship Enter-prises in fact turns out to show no profit for the year, itwill also turn out to be true that the company had toswitch suppliers during a large production run.20

The question asks:

The reasoning in the argument is most vulnerable to criti-cism on which one of the following grounds?

This question tells you that you should be looking fora problem with the argument. When you analyze theargument, you can identify the problem if you recognizethat there may well be other reasons for not showing aprofit besides having to switch suppliers in the middle ofa large production run. This points to a major oversightin the argument. At this point, you are ready to reviewthe answer choices.

One answer choice says:

The argument is a circular argument made up of an open-ing claim followed by a conclusion that merelyparaphrases that claim.

This gives a general account of an argument flaw, butclose inspection shows that the Blankenship argumentdoes not have this flaw. That argument’s conclusion sayssomething quite different from what was said in theargument’s premise. The conclusion says “If there is noprofit, then there was a switch in suppliers.” The prem-ise is superficially similar, but it says “If there is a switchin suppliers, then there will be no profit.” So this answerchoice is not a legitimate criticism.

Another answer choices reads:

The argument fails to establish that a condition underwhich a phenomenon is said to occur is the only conditionunder which that phenomenon occurs.

This is the correct answer. The argument could onlysucceed if it showed that switching suppliers in the mid-dle of a large production run is the only condition underwhich the company will show no profit for the year. Butthe argument fails to establish this point. Note that thisanswer choice points out what is wrong with this partic-ular argument using general terms that could covermany different arguments.

A Guide to Logical Reasoning Questions 35

20 December 1994 LSAT, section 2, question 26.

Cross-reference: This is an example of acommon reasoning error: confusion between“sufficient conditions” and “necessaryconditions.” A more detailed discussion ofsufficient and necessary conditions, and ofcorrect and incorrect ways of reasoning withconditions, can be found on pages 24–25.

Some Points to Consider

� When you begin a flawed reasoning question, youshould first try to get fairly clear about just where theargument goes wrong. If you have a reasonably clearsense of what is wrong with the reasoning, you canthen look for the answer choice that describes thekind of error you have identified. Keep in mind thatthe descriptions offered may be very general—onesthat could also apply to arguments very different insubject matter from the one you’re considering.

� Keep in mind that for an answer to be the correctanswer, it is not enough that it describe some aspectof the argument correctly. The correct answer mustdescribe a flaw in the reasoning of the particularargument under consideration.

� When dealing with a flawed argument that containsquantitative or statistical information, always checkto see that the reference groups mentioned in theargument are appropriate and consistent. An argumentmay, for example, present information about the per-centage of primary school teachers who have degrees ineducation and go on to draw a conclusion about the per-centage of people who receive degrees in education andgo on to teach primary school. Detecting this shift inreference groups is the key to identifying the flaw inthis argument. It is not unusual for argument flaws toinvolve subtle shifts such as this.

� Some flawed arguments involve errors in reasoningabout the relationship between proportions andquantities. Example 2 is a case in point. That argu-ment involves a shift from talk about proportions inthe premises to a statement about relative quantitiesin the conclusion. In some other argument, theremight be an illicit shift from quantities in the premisesto proportions in the conclusion. So pay close atten-tion to any shifts like these.

� Matching Argument Flaws

In your test you will sometimes encounter logical rea-soning questions like the following:

Which one of the following arguments is most similar inits flawed reasoning to the argument above?

It is best to approach these questions in a straightfor-ward way. First, try to determine in what way thereasoning in the reference argument is flawed. Then goover the arguments in the answer choices until you findthe one whose reasoning is flawed in just the same way.

The wording of the question tells you that the referenceargument is in fact flawed in its reasoning and that at leastone other argument is, too. There is no need to worry aboutthe precise wording of that flaw: the question does nothinge on it. But you do need to get a reasonably clear ideaof the kind of reasoning error committed, because youwill be making a judgment about whether one of theanswer choices matches that error.

For this type of question, incorrect answer choices canbe of two kinds. Some present arguments that are per-fectly good, so they don’t contain any flaw at all, letalone one that matches the one in the reference argu-ment. Others do present flawed arguments, but thereasoning errors in those arguments are clearly differentfrom the one in the reference argument.

An Example

Consider the following argument:

If the majority of the residents of the apartment complexcomplain that their apartments are infested with ants, thenthe management of the complex will have to engage the ser-vices of an exterminator. But the majority of the residents ofthe complex indicate that their apartments are virtually freeof ants. Therefore, the management of the complex will nothave to engage the services of an exterminator.21

The question you are asked about this argument is

Which one of the following arguments contains a flawedpattern of reasoning parallel to that contained in theargument above?

This question directs us to look for a flawed pattern ofreasoning in the reference argument and to look for ananswer choice that contains a similarly flawed pattern. Thismeans that we should look for an argument that takes thesame flawed approach to establishing its conclusion as the

A Guide to Logical Reasoning Questions36

21 June 1994 LSAT, section 4, question 20.

reference argument does, even though the two argumentsmight not share other characteristics, such as subject matter.

So what exactly is the flaw in the reference argument?One of the argument’s premises says that under a certaincondition the exterminator will have to come, and the sec-ond premise says that this condition is not met. Theargument concludes that the exterminator will not have tobe hired. But it is not difficult to see that something iswrong with this way of arguing: the problem is that theremay be other conditions under which the exterminator hasto be hired. To use a concrete example, it may also be truethat if there is a rodent infestation in the apartment com-plex, the management has to call in an exterminator. So thefact that the condition about ants is not met is not a goodenough reason for concluding that the exterminator willnot have to be hired. Without offering any reasons fordoing so, the argument treats one circumstance that wouldproduce a certain result as though it were the only cir-cumstance under which this result comes about.

The next step is to check the answer choices and to findthe one that exhibits the same flawed pattern of reason-ing. So let’s consider the following choice:

The number of flights operated by the airlines cannot bereduced unless the airlines can collect higher airfares. Butpeople will not pay higher airfares, so it is not the case thatthe number of flights will be reduced.

Is this an instance of the same flawed pattern? Thisargument is like the reference argument in that one of itspremises asserts that under a certain condition (airlinescannot collect higher airfares) something will happen(schedules will not be cut). But this argument is unlikethe reference argument in that its second premise actu-ally meets the condition set out in the first premise. Weare told that “people will not pay higher airfares,” so itstands to reason that airlines cannot collect higher air-fares. And thus the conclusion—that the number offlights will not be reduced—follows from these premises.So this argument does not exhibit the same pattern offlawed reasoning as the reference argument. In fact, itdoes not exhibit flawed reasoning at all.

Let’s try another one of the answer choices:

Most employees will attend the company picnic if theentertainment committee is successful in getting a certainband to play at the picnic. But that band will be out of thecountry on the day of the picnic, so it is not true that mostemployees will attend.

Is this an instance of the pattern you are trying tomatch? Again, there is a conditional statement: if a cer-

tain band plays at the picnic, most employees willattend. So again, under a certain condition, somethingwill happen. In this argument, the second premise indi-cates that the condition will not be met. The band, beingout of the country, will certainly not play at the picnic.The argument goes on to conclude that most employeeswon’t attend the picnic. So, as in the reference argument,this conclusion says that the thing that would happenunder the original condition will not happen.

This is an exact match of the pattern of reasoning in thereference argument, hence it is an exact match of theflaw. As with the reference argument, the flaw can beillustrated concretely. Suppose, for example, that if theentertainment committee hires a certain well-knowncomedian to perform at the picnic, most employees willattend. This shows that the conclusion could be trueeven if what the argument tells us about the band is true.

Some Points to Consider

� For this type of question, there is no need to decidewhether the reference argument is flawed. You aretold that there is a flawed pattern of reasoning under-lying the argument. It is sometimes useful, however,to determine whether or not an argument in ananswer choice is flawed. If such an argument is notflawed, it cannot be the correct answer, because it willnot be a relevant match for the reference argument.

� Remember that, although you need to get a reasonablyclear fix on the kind of reasoning error committed in thereference argument, you do not need to come up with aprecise formulation of that error. Since you are not askedto put your understanding of the reasoning error intowords, all you need is a solid intuitive grasp of wherethe reasoning goes wrong.

� With this type of question, you look for the argumentthat most closely matches the reference argument interms of its flawed reasoning. So other similaritieswith the reference argument, such as in the way theargument is expressed or in its subject matter, areirrelevant. Two arguments can be expressed in verydifferent ways, or be about very different things, eventhough they exhibit the same pattern of reasoning.

Cross-reference: For a more extensivediscussion of this point, see “Identifying theParts of an Argument” on pages 15–16.

A Guide to Logical Reasoning Questions 37

Explanations

Some of the questions in the Logical Reasoning sectionrequire you to identify a potential explanation for somestate of affairs that is described to you. Broadly speaking,these questions come in two types: one in which you needto find an explanation for one particular situation, andanother in which you need to explain how two seeminglyconflicting elements of a given situation can both be true.

In the first sort of case, the phenomenon to beexplained will merely be something that one would notordinarily expect, the kind of thing that makes peoplesay, “There must be an explanation for this.” Imagine,for example, that it is discovered that domestic cats withpurely gray coats are, on average, significantly heavierthan those with multicolored coats. A fact like this callsfor an explanation. The wording of a correspondingquestion would be along the lines of:

Which one of the following, if true, most helps to explainthe difference in average weights?

In the second sort of case, the phenomenon to beexplained is more complex. You are not simply presentedwith one fact that seems to require an explanation. Rather,you are presented with statements that appear to conflictwith one another, and your task is to identify the answerchoice that most helps to resolve this apparent discrep-ancy. That is, you are to select an answer choice thatexplains not just one or the other of the apparently con-flicting elements but explains how they can both be true.With this sort of question, the passage might say, forexample, that people spend much less time reading todaythan they did 50 years ago, and yet many more books aresold per year now than were sold 50 years ago. A typicalwording for this sort of question is:

Which one of the following, if true, most helps to resolvethe apparent discrepancy in the information given above?

Example 1

The situation that follows seems to call for an explana-tion. In this case, a software production company’sdecision to refrain from prosecuting people who illegallycopy its program raises questions regarding the com-pany’s reasons.

The company that produces XYZ, a computer spreadsheetprogram, estimates that millions of illegally reproducedcopies of XYZ are being used. If legally purchased, thisnumber of copies would have generated millions of dollars

in sales for the company, yet despite a company-wide effortto boost sales, the company has not taken available legalmeasures to prosecute those who have copied theprogram illegally.22

The question that is based on this situation readsas follows:

Which one of the following, if true, most helps to explainwhy the company has not taken available legal measures?

Incorrect answer choices such as

XYZ is very difficult to copy illegally, because a sophisti-cated anticopying mechanism in the program must firstbe disabled.

do nothing to help us understand the company’s deci-sion. They may, however, be relevant to some aspect ofthe situation. The answer choice above, for example,does suggest that those who do the illegal copying areknowledgeable about computers and computer software,but it doesn’t throw any light on the company’s decisionnot to prosecute.

The correct answer,

Many people who purchase a software program like XYZare willing to purchase that program only after they havealready used it.

on the other hand, does suggest a reason for the com-pany to tolerate the use of illegal copies of its program:those copies happen to serve as effective marketing aidsin many cases and lead to legal sales of the program. Thecompany may think that it has more to lose than to gainfrom going to court in order to stop the illegal copying.At the very least, the correct answer tells us that there isa disadvantage for the company in stopping the illegalcopying, and this helps to explain why no legal mea-sures are taken.

Example 2

Of the five bill collectors at Apex Collection Agency, Mr.Young has the highest rate of unsuccessful collections. YetMr. Young is the best bill collector on the agency’s staff.23

This situation has an air of paradox. It seems clear thata superior ability to bring a collection effort to a success-

A Guide to Logical Reasoning Questions38

22 December 1994 LSAT, section 4, question 2.23 June 1995 LSAT, section 2, question 5.

ful conclusion is what makes one bill collector betterthan another. So how can Mr. Young, who has the lowestrate of successful collections, be the best bill collector?This is the focus of the question that goes along withthis situation:

Which one of the following, if true, most helps to resolvethe apparent discrepancy?

Consider the following answer choice:

Mr. Young’s rate of collections per year has remainedfairly steady in the last few years.

This gives us information that is pertinent to Mr.Young’s performance as a bill collector. But it gives us noreason to think that Mr. Young could be the best bill col-lector at the agency despite having the lowest collectionrate. It only gives us more reason to think Mr. Young is apoor bill collector, because it allows us to infer that hiscollection rate has been low for years.

Now consider another choice:

Mr. Young is assigned the majority of the most difficultcases at the agency.

This gives us reason to think more highly of Mr.Young’s ability as a bill collector, because it makes senseto assign the most difficult cases to Mr. Young if he isvery good at collecting bills. And if his rate of success isrelatively low, this is not really a surprise, because hiscases tend to be more difficult. So this answer makes itclear how two facts that seemed to be difficult to recon-

cile with one another can in fact both be true. Thisresolves the apparent discrepancy in a satisfying way.

Some Points to Consider

� The correct answers to these questions do not gener-ally offer complete and detailed explanations. Rather,they present crucial information that plays an impor-tant part within an adequate explanation.

� Pay close attention to what you are asked to explain.In the case of simple explanations of a particular fac-tual matter, the wording of the question will directyou specifically to the fact to be explained. In the caseof explanations that resolve an apparent conflict,however, it is generally up to you to develop a clearpicture of that conflict.

� In most cases, there is more than one way to explain aset of facts or resolve an apparent conflict. So it is gen-erally not a good idea to work out several explanatorystories in your head before examining the answerchoices. Go to the answer choices instead and, foreach choice, determine whether it helps to explain orresolve the situation.

� Note that these questions are qualified by the expres-sion “if true.” This indicates that you do not have tobe concerned about the actual truth of the answerchoices. Simply consider each answer choice asthough it were true.

A Guide to Logical Reasoning Questions 39